Fourier Series for a Square-wave Function

Click For Summary
The discussion focuses on determining the Fourier series expansion for a square wave function defined by y(t) = h within a specific interval and zero elsewhere, with a period T = 2. The Fourier analysis coefficients are calculated, leading to a simplified expression for A_n as A_n = (h/πn)sin(πn). The significance of sin(πn) is highlighted, suggesting that it equals zero for integer values of n. Additionally, the transformation of y(t) into an odd function x(t) = y(t) - h/2 is mentioned, prompting a discussion on the implications for the Fourier series of odd functions. The relationship between the Fourier series of x and y is also questioned, emphasizing the properties of odd functions in Fourier analysis.
K.QMUL
Messages
54
Reaction score
0

Homework Statement



Consider the square wave function defined by y(t) = h (constant) when 0 ≤ (t + nT) ≤1,
y(t) = 0 elsewhere, where T = 2 is the period of the function. Determine the Fourier series
expansion for y(t).

Homework Equations



Fourier Analysis Coefficients

The Attempt at a Solution



Please look at the attachment, I am not convinced I have done it right so far... please help.
 

Attachments

  • 20131210_200639.jpg
    20131210_200639.jpg
    29.1 KB · Views: 500
Physics news on Phys.org
Your answer for ##A_n## can be simplified. Since ##T=2##, it becomes
$$A_n = \frac{h}{\pi n} \sin(\pi n)$$
Now what is ##\sin(\pi n)##?
 
By the way, you can save yourself some work as follows. Note that if we define ##x(t) = y(t) - h/2##, then ##x## is an odd function. What can you say about the Fourier series of an odd function? And how does the Fourier series of ##x## relate to the Fourier series of ##y##?
 
Question: A clock's minute hand has length 4 and its hour hand has length 3. What is the distance between the tips at the moment when it is increasing most rapidly?(Putnam Exam Question) Answer: Making assumption that both the hands moves at constant angular velocities, the answer is ## \sqrt{7} .## But don't you think this assumption is somewhat doubtful and wrong?

Similar threads

  • · Replies 3 ·
Replies
3
Views
2K
  • · Replies 1 ·
Replies
1
Views
2K
Replies
3
Views
2K
  • · Replies 2 ·
Replies
2
Views
2K
  • · Replies 8 ·
Replies
8
Views
3K
  • · Replies 4 ·
Replies
4
Views
1K
  • · Replies 6 ·
Replies
6
Views
2K
  • · Replies 6 ·
Replies
6
Views
2K
  • · Replies 16 ·
Replies
16
Views
2K
  • · Replies 1 ·
Replies
1
Views
2K